Các bất đẳng thức AG - Lồi, GG - lồi, GA - lồi và ứng dụng

Mục lục trang Mở đầu . . . . . . . . . . . . . . . . . . . . . . . . . . . . . . . . . . . . . . . . . . . . . . . . . . . . . . . . . . . . . . . 5 Chương 1. Bất đẳng thức lồi . . . . . . . . . . . . . . . . . . . . . . . . . . . . . . . . . . . . . . 7 1.1. Hàm lồi trong không gian Rn . . . . . . . . . . . . . . . . . . . . . . . . . . . . . . . . . . . . . .7 1.1.1. Định nghĩa . . . . . . . . . . . . . . . . . . . . . . . . . . . . . . . . . . . . . . . . . . . . . . . . . . . . . . 7 1

pdf32 trang | Chia sẻ: huyen82 | Lượt xem: 2105 | Lượt tải: 0download
Tóm tắt tài liệu Các bất đẳng thức AG - Lồi, GG - lồi, GA - lồi và ứng dụng, để xem tài liệu hoàn chỉnh bạn click vào nút DOWNLOAD ở trên
.1.2. Một số tính chất chung . . . . . . . . . . . . . . . . . . . . . . . . . . . . . . . . . . . . . . . . . . 7 1.1.3. Ví dụ . . . . . . . . . . . . . . . . . . . . . . . . . . . . . . . . . . . . . . . . . . . . . . . . . . . . . . . . . . 11 1.2. Hàm lồi một biến . . . . . . . . . . . . . . . . . . . . . . . . . . . . . . . . . . . . . . . . . . . . . . . . 16 1.2.1. Đặt vấn đề . . . . . . . . . . . . . . . . . . . . . . . . . . . . . . . . . . . . . . . . . . . . . . . . . . . . . 16 1.2.2. Các tính chất của hàm lồi một biến . . . . . . . . . . . . . . . . . . . . . . . . . . . . 16 Chương 2. Bất đẳng thức số học và hình học . . . . . . . . . . . . . . . . . . .19 2.1. Định nghĩa . . . . . . . . . . . . . . . . . . . . . . . . . . . . . . . . . . . . . . . . . . . . . . . . . . . . . . 19 2.1.1. Hàm AG-lồi . . . . . . . . . . . . . . . . . . . . . . . . . . . . . . . . . . . . . . . . . . . . . . . . . . . . 19 2.1.2. Hàm GG-lồi . . . . . . . . . . . . . . . . . . . . . . . . . . . . . . . . . . . . . . . . . . . . . . . . . . . .19 2.1.3. Hàm GA-lồi . . . . . . . . . . . . . . . . . . . . . . . . . . . . . . . . . . . . . . . . . . . . . . . . . . . . 19 2.2. Tính chất . . . . . . . . . . . . . . . . . . . . . . . . . . . . . . . . . . . . . . . . . . . . . . . . . . . . . . . . 20 2.3. Một số áp dụng . . . . . . . . . . . . . . . . . . . . . . . . . . . . . . . . . . . . . . . . . . . . . . . . . . 22 2.4. Một số ví dụ . . . . . . . . . . . . . . . . . . . . . . . . . . . . . . . . . . . . . . . . . . . . . . . . . . . . . 25 2.4.1. Bài toán 1 . . . . . . . . . . . . . . . . . . . . . . . . . . . . . . . . . . . . . . . . . . . . . . . . . . . . . 25 2.4.2. Bài toán 2 . . . . . . . . . . . . . . . . . . . . . . . . . . . . . . . . . . . . . . . . . . . . . . . . . . . . . 26 2.4.3. Bài toán 3 . . . . . . . . . . . . . . . . . . . . . . . . . . . . . . . . . . . . . . . . . . . . . . . . . . . . . 28 2.4.4. Bài toán 4 . . . . . . . . . . . . . . . . . . . . . . . . . . . . . . . . . . . . . . . . . . . . . . . . . . . . . 29 2.4.5. Bài toán 5 . . . . . . . . . . . . . . . . . . . . . . . . . . . . . . . . . . . . . . . . . . . . . . . . . . . . . 30 2.4.6. Bài toán 6 . . . . . . . . . . . . . . . . . . . . . . . . . . . . . . . . . . . . . . . . . . . . . . . . . . . . . 31 Kết luận . . . . . . . . . . . . . . . . . . . . . . . . . . . . . . . . . . . . . . . . . . . . . . . . . . . . . . . . . . . . 33 Tài liệu tham khảo . . . . . . . . . . . . . . . . . . . . . . . . . . . . . . . . . . . . . . . . . . . . . . . . 34 5Mở đầu Trong lý thuyết Giải tích lồi, chúng ta đã biết đến hàm lồi, với bất đẳng thức Jensen có ảnh hưởng lớn đến lý thuyết Bất đẳng thức. Trong những năm gần đây, nhiều nhà toán học đã quan tâm tới việc mở rộng một số bất đẳng thức khác, thông qua việc nghiên cứu hàm lồi số học (Arithmetic convex functiuon), hàm lồi hình học (Geometric convex functiuon). Các tác giả C.P. Niculescu, R.A. Satnoianu (xem [5], [6]) đã nhận được nhiều kết quả thú vị trong khi xem xét các hàm lồi như vậy. Xuất phát từ những kết quả mà một số tác giả đã tìm được, tôi thấy cần hiểu biết thêm và muốn được tiếp cận nó. Vì lý do đó, tôi lựa chọn đề tài Các bất đẳng thức AG-lồi, GG-lồi, GA-lồi và ứng dụng . Nội dung của luận văn bao gồm hai chương: Chương 1: Bất đẳng thức lồi. Trong chương này, chúng tôi trình bày các kiến thức cơ sở có liên quan đến bất đẳng thức lồi, đó là: Khái niệm hàm lồi trong không gian Rn, Bất đẳng thức Jensen, Hàm lồi 1 biến số và một số ứng dụng. Chương 2: Bất đẳng thức số học và hình học. Đây là nội dung chính của khoá luận. Trong chương này, sau khi trình bày bất đẳng thức số học và hình học, chúng tôi cố gắng xem xét một số tính chất và sự áp dụng của nó. Cuối cùng là 6 bài toán áp dụng nhằm minh hoạ cho ảnh hường của các bất đẳng thức đã nêu. Các kết quả của chương 2, chúng tôi chủ yếu dựa theo các kết quả của các bài báo [5], [6], có tham khảo thêm [1]. Khoá luận được thực hiện và hoàn thành dưới sự hướng dẫn khoa học của PGS. TS. Trần Xuân Sinh. Tác giả xin bày tỏ lòng biết ơn sâu sắc tới 6thầy về sự hướng dẫn tận tâm của thầy đối với tác giả trong suốt thời gian học tập và nghiên cứu. Nhân dịp này, tác giả xin gửi lời cảm ơn tới PGS. TS. Nguyễn Văn Quảng, các thầy cô giáo trong tổ Xác suất thống kê, khoa Toán, khoa Sau Đại học. Đồng thời, tác giả xin bày tỏ lòng biết ơn tới gia đình và các bạn bè, đã quan tâm, góp ý và tạo điều kiện giúp đỡ tác giả thực hiện khoá luận này. Mặc dù đã cố gắng song khoá luận không thể tránh khỏi những sai sót. Tác giả mong nhận được những đóng góp của quý thầy cô giáo và các bạn để khoá luận được hoàn thiện hơn. Tác giả xin chân thành cảm ơn! Vinh, tháng 05 năm 2009 Tác giả 7Chương 1 Bất đẳng thức lồi 1.1. Hàm lồi trong không gian Rn 1.1.1. Định nghĩa • Hàm f(k) xác định trên tập lồi M được gọi là hàm lồi, nếu với mọi x, y ∈ M và α ∈ [0, 1] có bất đẳng thức f [αx + (1− α)y] ≤ αf(x) + (1− α)f(y). (1) Ta gọi bất đẳng thức (1) là bất đẳng thức lồi. • Nếu f [αx + (1− α)y] ≥ αf(x) + (1− α)f(y) thì ta nói f là hàm lõm. Ví dụ:  Hàm tuyến tính và tuyến tính afin là hàm vừa lõm, vừa lồi.  Hàm y = max{x, a}, với a > 0 là hàm lồi (tuyến tính từng khúc với điểm gẫy là x = a).  Cho f(x) là hàm lồi, liên tục trên tập lồi M . Khi đó hàm y = max{f(x), 0}, với x ∈ M là hàm lồi. 1.1.2. Một số tính chất chung • Bất đẳng thức Jensen Định lý. Cho tập lồi M ⊂ R, xi ∈ M, i = 1, k, λi ≥ 0, ∑k i=1 λi = 1. Hàm f là hàm lồi trên tập lồi M khi và chỉ khi f ( k∑ i=1 λixi ) ≤ k∑ i=1 λif(xi). (2) 8Chứng minh. Giả sử có bất đẳng thức (2), khi đó với k = 2, theo định nghĩa thì f là hàm lồi. Ngược lại, giả sử f(x) là hàm lồi trên M . Xét x = k∑ i=1 λixi,∀λi ≥ 0, k∑ i=1 λi = 1, xi ∈ M. Rõ ràng do M lồi nên x ∈ M. Để chứng minh (2) ta quy nạp theo k . Với k = 2, bất đẳng thức đúng. . Giả sử bất đẳng thức đúng với k− 1, ta chứng minh đúng với k. Ta có k∑ i=1 λif(xi) = k−1∑ i=1 λif(xi) + λkf(xk). Không mất tính tổng quát, giả sử 0 < λk < 1. Khi đó k∑ i=1 λif(xi) = (1− λk) k−1∑ i=1 λi 1− λkf(xi) + λkf(xk) = λkf(xk) + (1− λk) k−1∑ i=1 βif(xi). với βi = λi 1−λk ≥ 0,∀i = 1, 2, ..., k − 1, rõ ràng ∑k−1 i=1 βi = 1. Do vậy, theo giả thiết quy nạp ta được k∑ i=1 λif(xi) ≥ (1− λk) k−1∑ i=1 βif(xi) + λkf(xk) ≥ f [λkxk + (1− λk) k−1∑ i=1 βixi] ≥ f ( k∑ i=1 λixi ) . Định lý được chứng minh. 2 • Một số hệ quả 1) Nếu hàm số y = f(x) lồi trên tập lồi M thì với mọi x1, x2, ..., xn ∈ M 9ta có bất đẳng thức f (x1 + x2 + ... + xn n ) ≤ f(x1) + f(x2) + ... + f(xn) n , (n ≥ 2). (3) 2) Nếu hàm số y = f(x) lồi trên M lồi thì với mọi x1, x2, ..., xn ∈ M ta có bất đẳng thức f (m1x1 + m2x2 + ... + mnxn m1 + m2 + ... + mn ) ≤ m1f(x1) + m2f(x2) + ... + mnf(xn) m1 + m2 + ... + mn . (4) Thật vậy, áp dụng bất đẳng thức Jensen (2) lấy với λ1 = λ2 = ... = λn = 1 n , ta có bất đẳng thức (3), lấy với λi = mi m1+m2+...+mn ,mi > 0, i = 1, 2, ..., n, ta có bất đẳng thức (4). 3) Hàm f liên tục trên tập M là hàm lồi khi và chỉ khi f (x + y 2 ) ≤ 1 2 (f(x) + f(y)). (5) Chứng minh. Điều kiện cần là rõ ràng vì từ bất đẳng thức lồi (1) đúng với mọi λ thuộc [0, 1]. Điều kiện đủ. Ta cần chứng minh nếu có (5) thì hàm liên tục f là lồi. Từ (5) ta có f (x(1) + x(2) + x(3) + x(4) 4 ) ≤ 1 4 [ f(x(1)) + f(x(2)) + f(x(3)) + f(x(4)) ] . Bằng quy nạp hoàn toàn với m = 2k, k là số tự nhiên nào đó, ta nhận được bất đẳng thức f ( 1 m m∑ i=1 x(i) ) ≤ 1 m m∑ i=1 f(x(i)). (∗) Ta chứng minh quy nạp bất đẳng thức (*) đúng với mọi số tự nhiên m. Rõ ràng với m = 2k đủ lớn thì (*) đúng. Gải sử với m = N bất đẳng thức đúng, tức là f ( 1 N N∑ i=1 x(i) ) ≤ 1 N N∑ i=1 f(x(i)). 10 Ta chứng minh bất đẳng thức đúng với m = N − 1. Đặt x(N) = 1 N − 1(x (1) + x(2) + ... + x(N−1)), ta có x(N) = 1 N (x(1) + x(2) + ... + x(N)), và f(x(N)) = f ( N∑ i=1 x(i) ) ≤ 1 N N∑ i=1 f(x(i)) = 1 N N−1∑ i=1 f(x(i)) + 1 N f(x(N)). Từ đó ta được f (N−1∑ i=1 x(i) ) ≤ 1 N − 1 N−1∑ i=1 f(x(i)). Như vậy (*) đúng với mọi m số tự nhiên. Để chứng minh f lồi, ta phải chứng minh f thỏa mãn bất đẳng thức lồi (1). Lấy x, y ∈ M và k,m là các số tự nhiên, k < m. Từ (*) ta có f ( kx + (m− k)y m ) = f ( k m x + m− k m y ) ≤ 1 m [ kf(x) + (m− k)f(y)]. Đặt λ = km , ta có λ là số hữu tỉ thỏa mãn 0 < λ < 1 và f(λx + (1− λ)y) ≤ λf(x) + (1− λ)f(y). Vì f là hàm liên tục, do vậy bất đẳng thức lồi trên cũng đúng với mọi giá trị thực λ thuộc đoạn [0, 1].  4) Cho M là tập hợp lồi, f(x) là hàm liên tục lồi và xác định không âm trên M , khi đó f 2(x) cũng là hàm lồi trên M . Thật vậy, do M lồi nên x, y thuộc M thì x+y2 thuộc M . Lại do f lồi nên theo (5) thì f (x + y 2 ) ≤ 1 2 [f(x) + f(y)]. 11 Do f là hàm không âm nên f 2 (x + y 2 ) ≤ 1 4 [f(x) + f(y)]2. Mặt khác ta có 1 4 [f(x) + f(y)]2 − 1 2 f 2(x)− 1 2 f 2(y) = −1 4 [f 2(x) + f 2(y)]. Nghĩa là f 2(x) là hàm lồi trên M.  5) Tổng hữu hạn các hàm lồi xác định trên tập lồi M là hàm lồi. Chú ý rằng, hiệu hai hàm lồi nói chung không phải là hàm lồi. 6) Cho f(x) là hàm lồi trên tập lồi M và số thực α cố định. Khi đó tập Mα = {x ∈ Rn : f(x) ≤ α} là tập lồi. Tổng quát ta có: Cho fi, i = 1, 2, ..., k là các hàm lồi xác định trên tập lồi M . Khi đó tập hợp Mα = {x ∈ M : fi(x) ≤ αi, i = 1, 2, ..., k, αi ∈ R} là tập lồi. 1.1.3. Ví dụ a) Chứng minh các bất đẳng thức số học • Bất đẳng thức Côsi a1 + a2 + ... + an n ≥ n√a1a2...an. Giải: Ta có hàm −lnx là hàm lồi, áp dụng bất đẳng thức Jensen dạng đơn giản f (x1 + x2 + ... + xn n ) ≤ f(x1) + f(x2) + ... + f(xn) n 12 Ta có −lna1 + a2 + ... + an n ≥ ln(a1a2...an) n ⇔ a1 + a2 + ... + an n ≥ n√a1a2...an .  • Bất đẳng thức Bunhiacôpxki (a1b1 + a2b2 + ... + anbn) 2 ≤ (a21 + a22 + ... + a2n)(b21 + b22 + ... + b2n). trong đó a1, a2, ..., an; b1, b2, ..., bn là 2n số tuỳ ý. Giải: Xét hàm lồi f(x) = x2 với mọi x ∈ R áp dụng bất đẳng thức Jensen dạng tổng quát ta có (m1x1 + m2x2 + ... + mnxn m1 + m2 + ... + mn )2 ≤ m1x 2 1 + m2x 2 2 + ... + mnx 2 n m1 + m2 + ... + mn ⇔ (m1x1+m2x2+...+mnxn)2 ≤ (m1+m2+...+mn)(m1x21+m2x22+...+mnx2n). Đặt mi = b 2 i và xi = ai bi ta có( b21 a1 b1 + b22 a2 b2 + ... + b2n an bn ) ≤ (b21 + b22 + ... + b2n) ( b21 a21 b21 + b22 a22 b22 + ... + b2n a2n b2n ) ⇔ (a1b1 + a2b2 + ... + anbn) ≤ (a21 + a22 + ... + a2n)(b21 + b22 + ... + b2n). • Bất đẳng thức Mincôpxki n √ a1a2...an + n √ b1b2...bn ≤ n √ (a1 + b1)(a2 + b2) + ... + (an + bn) trong đó a1, a2, ..., an; b1, b2, ..., bn là 2n số dương. Giải: Xét hàm số f(x) = ln(1 + ex). Khi đó f ′(x) = ex 1 + ex , f”(x) = ex (1 + ex)2 > 0,∀x ∈ R. Vì vậy f(x) là hàm lồi trên R. áp dụng bất đẳng thức Jensen dạng đơn giản f (x1 + x2 + ... + xn n ) = f(x1) + f(x2) + ... + f(xn) n . 13 Chọn xi = ln bi ai , i = 1, 2, ..., n ta có ln ( 1 + e (ln b1 a1 +...+ln bnan ) n ) ≤ ln(1 + eln b1a1 ) + ... + ln(1 + eln bnan ) ⇔ ln ( 1 + e (ln b1 a1 +...+ln bnan ) n ) ≤ ln(1 + e ln b1 a1 ) + ... + ln(1 + eln bn an ) n ⇔ n √ a1a2...an + n √ b1b2...bn n √ a1a2...an ≤ n √ (a1 + b1)(a2 + b2)...(an + bn) a1a2...an ⇔ n√a1a2...an + n √ b1b2...bn ≤ n √ (a1 + b1)(a2 + b2) + ... + (an + bn).  • Cho n > 1 và x1, ..., xk > 0. Chứng minh rằng xn1 + x n 2 + ... + x n k k ≥ (x1 + x2 + ... + xk k )n . Giải: Xét hàm lồi f(x) = xn (x > 0) áp dụng bất đẳng thức Jensen đơn giản f (x1 + x2 + ... + xk k ) ≤ f(x1) + f(x2) + ... + f(xk) k . Ta có (x1 + x2 + ... + xk k )n ≤ x n 1 + x n 2 + ... + x n k k .  b) Chứng minh các bất đẳng thức hình học • Cho A,B,C là 3 góc của một tam giác. Chứng minh sinA sinB sinC ≤ 3 √ 3 8 . Giải: Do A,B,C là ba góc của một tam giác nên sinA, sinB, sinC > 0. áp dụng bất đẳng thức Cauchy cho ba số dương ta có sinA + sinB + sinC 3 ≥ 3 √ sinA sinB sinC ⇔ sinA× sinB × sinC ≤ (sinA + sinB + sinC 3 )3 ≤ ( 3√3 2 3 )3 = 3 √ 3 8 . 14 (ở đây ta đã sử dụng kết quả sinA + sinB + sinC ≤ 3 √ 3 2 ). • cosA + cosB + cosC ≤ 32 . Ta biết rằng trong một tam giác có ít nhất 1 góc nhỏ hơn 900. Không mất tính tổng quát ta giả sử 0 < A < pi2 . Đặt B′ = C ′ = B+C2 . Ta có A,B ′, C ′ ∈ (0, pi2 ). Hàm số f(x) = − cosx, và x ∈ (0, pi2 ) là hàm lồi. Do vậy cosA+cosB′+cosC ′ ≤ 3 cos A + B ′ + C ′ 3 = 3 cos A 3 = 3 2 (vì B′+C ′ = B+C). Mặt khác ta có cosB + cosC = 2 cos B + C 2 cos B − C 2 ≤ 2 cos B + C 2 = cosB′ + cosC ′. Do đó cosA + cosB + cosC ≤ cosA + cosB′ + cosC ′ ≤ 3 2 . Vậy cosA + cosB + cosC ≤ 3 2 . • sin A2 sin B2 sin C2 ≤ 18 . Giải: Xét hàm lõm f(x) = sinx x ∈ (0, pi). áp dụng bất đẳng thức Jensen đối với hàm lõm sin x ta có sin ( A 2 + B 2 + C 2 3 ) ≥ sin A 2 + sin B 2 + sin C 2 3 . Do sin A2 , sin B 2 , sin C 2 > 0, nên áp dụng bất đẳng thức Cauchy ta có sin A 2 + sin B 2 + sin C 2 ≥ 3 √ sin A 2 sin B 2 sin C 2 . 15 Từ đó suy ra sin A 2 sin B 2 sin C 2 ≤ (sin A2 + sin B2 + sin C2 3 )3 ≤ ( sin ( A 2 + B 2 + C 2 3 ))3 = (1 2 )3 = 1 8 . Vậy sin A2 sin B 2 sin C 2 ≤ 18 . • Chứng minh rằng trong mọi tam giác ABC ta luôn có( 1 + 1 sinA )( 1 + 1 sinB )( 1 + 1 sinC ) ≥ ( 1 + 2√ 3 )3 . Giải: Xét vế trái của bất đẳng thức trên ta có V T = 1 + 1 sinA + 1 sinB + 1 sinC + 1 sinA sinB + 1 sinB sinC + + 1 sinA sinC + 1 sinA sinB sinC . áp dụng bất đẳng thức Cauchy cho 3 số dương sinA, sinB, sinC, ta có sinA sinB sinC ≤ (sinA + sinB + sinC 3 )3 . Trên đoạn [0, pi] : sinx là một hàm lõm nên áp dụng bất đẳng thức Jensen đối với hàm lõm ta có sinA + sinB + sinC 3 ≤ sin (A + B + C 3 ) = sin pi 3 = √ 3 2 . Vậy sinA sinB sinC ≤ (√3 2 )3 ⇔ 1 sinA sinB sinC ≥ ( 2√ 3 )3 . Lại áp dụng bất đẳng thức Cauchy ta có 1 sinA + 1 sinB + 1 sinC ≥ 3 3 √ 1 sinA sinB sinC ≥ 3 ( 2√ 3 ) . 1 sinA sinB + 1 sinB sinC + 1 sinA sinC ≥ 3 3 √( 1 sinA sinB sinC )2 ≥ 3 ( 2√ 3 )2 . 16 Vậy ( 1 + 1 sinA )( 1 + 1 sinB )( 1 + 1 sinC ) ≥ 1 + 3 ( 2√ 3 ) + 3 ( 2√ 3 )2 + ( 2√ 3 )3 = 3 ( 1 + 2√ 3 )3 . 1.2. Hàm lồi một biến 1.2.1. Đặt vấn đề. Hàm lồi 1 biến có nhiều ứng dụng trong các bài toán sơ cấp và lý thuyết bất đẳng thức. Vì vậy, ngoài các tính chất chung như đã nêu trong mục 1.1.2, trong mục này chúng tôi cố gắng trình bày thêm một số kết quả đặc trưng đối với hàm lồi 1 biến. Cho f(x) là hàm lồi xác định trên tập lồi M , lấy x, y cố định thuộc M . Xét z = λx + (1− λ)y, 0 ≤ λ ≤ 1 thì z ∈ M. Ký hiệu φ(λ) = f(λx + (1− λ)y). Khi đó φ(λ) là hàm lồi 1 biến. 1.2.2. Các tính chất của hàm lồi một biến Định lý 1.1. Nếu f(x) là hàm lồi xác định trên tập lồi M thì φ(λ) là hàm lồi xác định trên đoạn [0, 1]. Định lý 1.2. Cho hàm lồi một biến f xác định trên [a, b]. Khi đó ta có f(t)− f(s) t− s ≤ f(u)− f(s) u− s ≤ f(u)− f(t) u− t trong đó u, s ∈ [a, b], s < t < u. Chứng minh. Từ f lồi, ta đặt λ = t− s u− s, t = u u− t u− s + s t− s u− s, s < t < u. 17 Ta có f(t) ≤ t− s u− sf(u) + s− t u− sf(s), s < t < u. Hay tương đương f(t)− f(s) ≤ t− s u− sf(u) + s− t u− sf(s) = s− t u− s [f(u)− f(s)]. Bất đẳng thức bên trái được chứng minh. Tương tự ta cũng có bất đẳng thức bên phải.  Định lý 1.3. Cho hàm số f(x) xác định và liên tục có đạo hàm cấp 2 trên (a, b). Khi đó f(x) lồi trên (a, b) nếu và chỉ nếu f”(x) ≥ 0,∀x ∈ (a, b). Chứng minh. Điều kiện đủ. Sử dụng định lý bất đẳng thức lồi Jensen, lấy x, z ∈ (a, b), (x < z). Ký hiệu c = x+z2 . Ta phải chứng minh f(c) ≤ 1 2 ( f(x) + f(z) ) . Xét đoạn thẳng [x, c]. Theo định lý Lagrăng, tồn tại ξ1 ∈ [x, c] sao cho f(c)− f(x) = f ′(ξ1).(c− x). Từ đó suy ra f ′(ξ1) = f(c)− f(x) c− x = f(c)− f(x) x+y 2 − x = f(c)− f(x) 1 2(z − x) . (1a) Lại xét đoạn [c, z]. Cũng theo định lý Lagrăng tồn tại ξ2 ∈ [c, z] sao cho f ′(ξ2) = f(z)− f(c) z − c = f(z)− f(c) z − x+y2 = f(z)− f(c) 1 2(z − x) . (1b) Theo giả thiết f”(x) ≥ 0, với mọi x ∈ (a, b), cho nên f ′(x) là hàm đồng biến trên [a, b], ta suy ra với ξ1 < ξ2 thì f ′(ξ1) < f ′(ξ2). (1c) Từ (1a), (1b), (1c) suy ra f(c)− f(x) 1 2(z − x) < f(z)− f(c) 1 2(z − x) . (1d) 18 Do z − c > 0 nên từ (1d) ta được 1 2 ( f(c)− f(x)) < 1 2 ( f(z)− f(c)), tức là f(c) < 1 2 ( f(z) + f(x) ) . Điều kiện cần. Người ta đã chứng minh rằng nếu f lồi trên [a, b] thì f ′ không giảm trên [a, b]. Từ đó suy ra f” ≥ 0. Đó là điều phải chứng minh. 2 • Hệ quả. Các hàm số sau đây là hàm lồi y = xk, với x > 0, k = 1, 2, ... y = ax, với a > 1 y = − loga x, với a > 1, x > 0 y = sin x, x ∈ (pi, 2pi). 19 Chương 2 Bất đẳng thức số học và hình học 2.1. Định nghĩa. Chương 1, chúng ta đã xét tới hàm lồi, mà trọng tâm là bất đẳng thức Jensen cùng những ứng dụng của nó. Tìm tòi những bất đẳng thức mới, tác giả C.P. Niculescu, R.A. Satnoianu (xem [5], [6]) đã đưa ra các khái niệm hàm lồi số học (Arithmetic convex functiuon), viết tắt là hàm A-lồi, hoặc hàm lồi Hình học (Geometric convex functiuon), viết tắt là hàm G-lồi. Sự kết hợp tính chất số học và hình học, sẽ cho ta các hàm AA-lồi, hàm AG-lồi, hàm GA-lồi, hàm GG-lồi. Theo các tác giả thì coi hàm AA-lồi là hàm luôn luôn lồi. Trong các trường hợp khác sẽ được định nghĩa như sau: Cho tập hợp I ⊂ (0,+∞) và hàm số f : I → (0,+∞). 2.1.1. Hàm AG-lồi. Hàm f được gọi là hàm AG-lồi (hoặc là hàm log-lồi) nếu với mọi x, y ∈ I, 0 ≤ α ≤ 1 thì f((1− α)x + αy) ≤ f (1−α)(x).fα(y). (2.1) Điều đó tương đương với hàm log f là lồi số học thông thường. 2.1.2. Hàm GG-lồi. Hàm f được gọi là hàm GG-lồi (hoặc là hàm nhân lồi) nếu với mọi x, y ∈ I, 0 ≤ α ≤ 1 thì f(x(1−α).yα) ≤ f (1−α)(x).fα(y). (2.2) 2.1.3. Hàm GA-lồi. Hàm f gọi là hàm GA-lồi nếu với mọi x, y ∈ I, 0 ≤ α ≤ 1 thì f(x(1−α).yα) ≤ (1− α)f(x) + αf(y). (2.3) 20 2.2. Tính chất Người ta đã chứng minh rằng: Nếu hàm số f : I → (0,+∞) thoả mãn x2f”(x) + xf ′(x) ≥ 0. (2.4) thì nó là hàm GA-lồi. Trong trường hợp (2.4) không thoả mãn thì sao. Tiếp theo chúng ta sẽ nghiên cứu một phần câu trả lời cho vấn đề được đặt ra. Giả thiết rằng f : (0,+∞) → (0,+∞) là hàm có vi phân bậc hai thỏa mãn f”(x) ≥ 0 trên miền xác định. Đặt g : (0,+∞) → (0,+∞), với g(x) = x2f”(x) + xf ′(x). Bất đẳng thức (2.4) không thoả mãn, nghĩa là tồn tại số thực r : 0 < r < 1, g(r) = 0 sao cho g < 0 trên (0, r) và g ≥ 0 trên (r,∞). Xét hàm h : (0,+∞)n → (0,+∞) xác định bởi h(x1, x2, ..., xn) = n∑ k=1 f(xk)− nf(1), với mọi x1, x2, ..., xn > 0 thoả mãn n∏ k=1 xk = 1. (2.5) Ta cũng giả thiết rằng các toạ độ của điểm tới hạn của h thoả mãn (2.5) có thể lấy 2 giá trị khác nhau, nghĩa là tồn tại a = b sao cho x1, x2, ..., xn = a, b tại điểm tới hạn của hợp thành (x1, x2, ..., xn) của h. Định lý 2.1. Nếu tất cả các điều kiện đã nêu được thoả mãn và mỗi k = 1, 2, ..., n, chúng ta có lim xk→0 h(x1, x2, ..., xn) ≥ 0, (2.6) vậy h(x1, x2, ..., xn) ≥ 0, với mọi x1, x2, ..., xn ≥ 0, thoả mãn ∏n k=1 xk = 1. Chứng minh. Trước hết ta chú ý rằng h là một hàm liên tục, xác định trên tập bị chặn (từ điều kiện), nên tồn tại giá trị m > −∞ sao cho h ≥ m, 21 cho mọi x1, ..., xn > 0, với ∏n k=1 xk = 1. Theo yêu cầu của định lý, chúng ta cần chỉ ra rằng m ≥ 0. Ngoài ra, từ (2.6) tất yếu suy ra tính bị chặn của miền xác định, nghĩa là có giới hạn khi xk → 0, với k = 1, ..., n. Đặt K = { (x1, x2, ..., xn) : x1, x2, ..., xn > 0, n∏ k=1 xk = 1 } . Việc còn lại chứng minh là ta cần thiết lập sự xác định phần trong của K. Chúng ta xét điểm cực trị của h, được tìm từ các điểm tới hạn. Theo giả thiết, các toạ độ của chúng có thể lấy 2 giá trị khác nhau. Điều đó suy ra rằng ∂h ∂xi = 0, i = 1, ..., n ⇒ {x01, x02, ..., x0n} = {a, b} tại tất cả các điểm tới hạn. Vì tính đối xứng, chúng ta có thể giả thiết rằng x01 ≤ x02 ≤ ... ≤ x0n và a ≤ b. Do vậy tồn tại q, với 1 ≤ q ≤ n, sao cho x01 = x02 = ... = x0q−1 = a và x0q = x 0 q+1 = ... = x 0 n = b (khi q = 1, chúng ta quy ước x 0 0 = 0). Để ý rằng theo (2.5) suy ra b ≥ 1. Chú ý rằng nếu q = 1 vậy thì không cần chứng minh vì trong trường hợp này kết luận được chỉ ra bởi điều kiện (2.4) cho ta điểm cực tiểu (hoặc suy ra trực tiếp từ (2.5)). Xét tiếp điều kiện h1(a, b) = (q − 1)f(a) + (n− q + 1)f(b)− nf(1). (2.7) Chú ý rằng theo (2.5) chúng ta có aq−1bn−q+1 = 1. (2.8) Chúng ta sẽ chỉ ra rằng h1(a, b) ≥ 0, với mọi a, b > 0 thoả mãn (2.8). Theo (2.8) điều này tương đương với h1(b) = (q − 1)f ( b n+1−q 1−q ) + (n− q + 1)f(b)− nf(1) ≥ 0. (2.9) 22 Tính toán cho thấy h′1(b) = 0 và h ′ 1(b) = h ′ 1 ( b n+1−q 1−q ) b n 1−q . Bởi vì b ≥ 1 và f ′ là hàm tăng nên chỉ khi b = 1 thì (2.9) trở thành đẳng thức. Ngoài ra, h”1(1) = n q−1(f”(1) + f ′(1)) ≥ 0 được chỉ ra bởi điều kiện (2.4) được sử dụng từ g tại x = 1 và r < 1. Điều đó chỉ ra rằng b0 = 1 là điểm cực tiểu đối với h1 và đẳng thức (2.9) là đúng tại điểm này. Điều đó cho thấy nó đúng với mọi điểm khác b ≥ 1. Cuối cùng, điều đó cho ta sự đúng đắn tại điểm cực tiểu của f và nó cũng đúng với tất cả điểm trong của miền xác định K. Chúng ta có thể trực tiếp thử lại các điểm trên biên (điểm giới hạn) của K cho thấy kết quả tương tự. Định lý chứng minh xong. 2 2.3. Một số áp dụng Trong tài liệu tham khảo [5], các tác giả đã chứng minh được bất đẳng thức n∑ i=1 ( (1 + λ) xn−1i xn−1i + λΠk 6=i xk ) 1 n−1 ≥ n, (2.10) với mọi n ≥ 1, xi > 0, i = 1, 2, ..., n và λ ≥ nn−1 − 1. Bây giờ ta đặt yi = Πk 6=i xk xn−1i , i = 1, 2, ..., n, ta có định lý sau đây: Định lý 2.2. Ta có n∑ i=1 1 n−1√1 + λyi ≥ n n−1√1 + λ (2.11) với mọi n ≥ 1 và yi > 0, i = 1, 2, ..., n, Πni=1yi = 1, λ ≥ nn−1 − 1. Chứng minh. Trong trường hợp n = 1, 2 thì bất đẳng thức (2.11) đúng. Ta xét với n > 2. 23 Với mỗi n > 1 và x, λ > 0, giả sử rằng f(x) = (1 + λx)−1/(n−1). Khi đó f ′(x) = −λ(n− 1)−1(1 + λx)−n/(n−1) < 0, f”(x) = λ2n(n− 1)−2(1 + λx)−(2n−1)/(n−1) > 0, với mọi x > 0 và với mỗi λ > 0, n > 1. Điều này cho thấy f là hàm giảm và lồi theo biến x. Với mỗi λ > 0, ta có x2f”(x) + xf ′(x) = λx (1 + λx)(1−2n)/(n−1) λ2 (1 + λx− n). Khi đó x2f”(x)+ xf ′(x) ≥ 0 khi 1+λx−n > 0. Điều đó xảy ra khi và chỉ khi x > n−1λ . Do vậy trên khoảng J = ( n−1 λ ,∞ ) hàm f là hàm GA-lồi. Từ giả thiết đã cho ta có n∏ i=1 yi = 1. (2.12) Vì thế (2.11) trở thành hh(y1, y2, ..., yn) = n∑ k=1 f(yi)− nf(1) ≥ 0, (2.13) với mọi yi > 0, i = 1, 2, ..., n thoả mãn (2.13) và λ ≥ nn−1 − 1. Từ đó ta nhận thấy rằng các điểm tới hạn của hàm hh thoả mãn điều kiện (2.12) phải thoả mãn đẳng thức d(y1) = d(y2) = ... = d(yn), trong đó d(y) = y(1+λy)n/(n−1) . Lúc này d là hàm đơn điều ngặt trên J và R−J , nên ta suy ra rằng các điểm tới hạn của hàm hh trong (2.13) có thể đạt được 2 giá trị a, b (a ≤ b). Ngoài ra (2.12) cho ta b ≥ 1. Tới đây, chúng ta thấy rằng có thể có ngoại lệ của điều kiện (2.6). Tất cả các giả thiết của định lý 2.1 được thoả mãn trong trường hợp chúng ta 24 đang xét, và chúng được kết thúc với tất cả các điểm trong của miền xác định. Bây giờ chúng ta xét tới trường hợp miền xác định của hàm hh trong (2.13) khi a → 0, (hoặc tương đương là khi b → 0). Bởi vì lim a→0 f(a) = 1, lim b→∞ f(b) = 0, nên chúng ta có q − 1 ≥ nf(1) = n(1 + λx)−1/(n−1). Điều đó là hiến nhiên đúng với điều kiện λ ≥ nn−1 − 1. Đẳng thức xảy ra với q = 2 và λ = nn−1 − 1. Từ đó suy ra rằng giả thiết (2.2) cố định trong trường hợp của chúng ta. Bởi vậy bất đẳng thức (2.13) thoả mãn với tất cả các điểm tới hạn yi > 0, i = 1, 2, ..., n và λ ≥ nn−1 − 1. Sử dụng định lý 2.1, chúng ta được điều phải chứng minh. 2 Định lý 2.3. Với mỗi α, β > 0, n ≥ 1 sao cho β ≥ (nn−1 − 1)α, chúng ta có bất đẳng thức n∑ i=1 ( xn−1i αxn−1i + βΠk 6=ixk ) 1 n−1 ≥ n(α + β) −1n−1 . (2.14) Chứng minh. Việc chứng minh định lý này là dễ dàng được suy ra từ định lý 2.1, thông qua lý luận như đã làm với định lý 2.2. 2 Từ định lý 2.1, ta cũng có định lý 2.4 sau đây: Định lý 2.4. Với mọi n ≥ 1, p ≥ 1, yi > 0, i = 1, 2, ..., n sao cho Πni=1yi = 1 và λ ≥ np − 1, chúng ta có bất đẳng thức n∑ i=1 (1 + λyi) −1/p ≥ n(α + β)−1/p. (2.15) Chứng minh. Việc chứng minh định lý 2.4 cũng thực hiện như định lý 2.2 bằng cách thay n− 1 bởi p. 25 Từ định lý 2.2 và định lý 2.4, chúng ta có được định lý mà công thức đối ngẫu với công thức (2.10) như sau: Định lý 2.5. Với mọi n ≥ 1, p ≥ 1, xi > 0, i = 1, 2, ..., n;α, β > 0;λ ≥ np − 1, chúng ta có bất đẳng thức n∑ i=1 ( (α + λβ) Πk 6=ixk Πk 6=ixk + λxn−1i )−1/p ≥ n. (2.16) Chứng minh. Bất đẳng thức (2.16) được suy ra từ (2.14) và (2.15) khi thay xi bởi 1/xi, i = 1, 2, ..., n. 2 Hệ quả. Nếu α, β > 0 thoả mãn β ≥ (nn−1 − 1)α thì n∑ j=1 (α + βxji ) −1 n−1 ≥ n(α + β) −1n−1 , (2.17) với mọi n ≥ 1, xi > 0, i = 1, 2, ..., n, sao cho Πni=1xi = 1. Chứng minh. Từ (2.16) thay p = n − 1, rồi nhân cả 2 vế của bất đẳng thức với (α + β)−1/(n−1), thực hiện các phép biến đổi ta được điều phải chứng minh. 2 2.4. Một số ví dụ Trong mục này, chúng ta xét một số bài toán đặc biệt như sau: 2.4.1. Bài toán 1. Chứng minh rằng a√ a2 + 8bc + b√ b2 + 8ca + c√ c2 + 8ab ≥ 1, với mọi số thực dương a, b, c. Chứng minh. Ta sẽ chọn số dương r sao cho a√ a2 + 8bc ≥ a 2r a2r + 2(bc)r . Bất đẳng thức này tương đương với a2(a2r + 2brcr)2 ≥ a4r(a2 + 8bc). 26 Điều đó xảy ra khi và chỉ khi b2rc2r + a2rbrcr ≥ 2a4r−2bc. Theo bất đẳng thức Côsi thì b2rc2r + a2rbrcr ≥ 2arb3r/2c3r/2. Ta chí cần chọn r sao cho  4r − 2 = r 3r/2 = 1. Từ đó suy ra r = 2/3. Khi đó theo bất đẳng thức AM-GM ta được a√ a2 + 8bc + b√ b2 + 8ca + c√ c2 + 8ab ≥ ≥ a 2r a2r + 2brcr + b2r b2r + 2arcr + c2r c2r + 2arbr ≥ a 2r a2r + b2r + c2r + b2r a2r + b2r + c2r + c2r a2r + b2r + c2r = 1. Đó là điều phải chứng minh. 2 Chú ý 1) Từ bất đẳng thức nêu trên, ta có đẳng thức xảy ra khi và chỉ khi a = b = c. 2) Phát biểu tổng quát cho bài toán này là: Đối với mọi a, b, c > 0 và λ ≥ 8 thì ta có bất đẳng thức a√ a2 + λbc + b√ b2 + λca + c√ c2 + λab ≥ 3√ 1 + λ . 2.4.2. Bài toán 2. Chứng minh rằng nếu a, b, c là các số thực dương thì bất đẳng thức sau luôn luôn đúng 1 a3(1 + b3) + 1 b3(1 + c3) + 1 c3(1 + a3) ≥ 3 abc(1 + abc) . 27 Chứng minh. Ta có a3b3c3 a3(b3 + 1) + 1 = a3b3c3 + 1 + a3b3 + a3 + 1 a3(b3 + 1) = a3b3(1 + c3) + a3 + 1 a3(b3 + 1) = b3(1 + c3) 1 + b3) + a3 + 1 a3(1 + b3) . Vậy theo bất đẳng thức AM-GM thì 3 + (a3b3c3 + 1) ( 1 a3(1 + b3) + 1 b3(1 + c3) + 1 c3(1 + a3) ) = = ∑ cyc a3 + 1 a3(1 + b3) + ∑ cyc b3(1 + c3) 1 + b3 ≥ 3 abc + 3abc, trong đó ∑ cyc a3 + 1 a3(1 + b3) = a3 + 1 a3(1 + b3) + b3 + 1 b3(1 + c3) + c3 + 1 c3(1 + a3)∑ cyc b3(1 + c3) 1 + b3 = b3(1 + c3) 1 + b3 + c3(1 + a3) 1 + c3 + a3(1 + b3) 1 + a3 . Mặt khác ta có a3b3c3 + 1 abc(1 + abc) ≤ a 3b3c3 + 1 abc = a2b2c2 + 1 abc ≤ 1 abc + abc− 1 = 1 + a2b2c2 − abc abc . Từ đó ta có 1 a3(1 + b3) + 1 b3(1 + c3) + 1 c3(1 + a3) ≥ 3(1 + a 2b2c2 − abc) a3b3c3 + 1 ≥ 3(1 + a 2b2c2 − abc)[ abc(1 + abc) ][ 1 + a2b2c2 − abc] = 3 abc(1 + abc) . 28 Đó là điều phải chứng minh. 2 Bất đẳng thức nêu trong bài toán 2, xảy ra đẳng thức đúng khi và chỉ khi a = b = c = 1. 2.4.3. Bài toán 3. Giả sử a1, a2, ..., an là các số dương, với a1a2...an = 1. Chứng minh rằng 1√ 1 + (n2 − 1)a1 + 1√ 1 + (n2 − 1)a2 + ... + 1√ 1 + (n2 − 1)an > 1. Chứng minh. Đặt xi = 1√ 1 + (n2 − 1)ai ,∀i = 1, n; p = x1x2...xn. Ta chứng minh bằng phản chứng. Giả sử x1 + x2 + ... + xn < 1. Khi đó ai = ( 1 x2i − 1) ) . 1 n2 − 1 = 1− x2i (n2 − 1)x2i . Từ đó suy ra n∏ i=1 (1− x2i ) = (n2 − 1)np2. (∗) Vì ∑n i=1 xi < 1, nên theo bất đẳng thức Côsi ta có 1− xj > −xj + n∑ i=1 xi ≥ (n− 1) n−1 √ p xj , 1 + xj ≥ xj + n∑ i=1 xi ≥ (n + 1) n+1√pxj. Lấy j = 1, 2, ..., n, rồi nhân tất cả các bất đẳng thức trên ta được n∏ i=1 (1− x2i ) > (n2 − 1)np2. Điều này mâu thuẫn với (*). Vậy ta có điều phải chứng minh. 2 Bất đẳng thức nêu trong bài toán 3 sẽ trở thành đẳng thức khi và chỉ khi a1 = a2 = ... = an = 1. 29 2.4.4. Bài toán 4. Cho a1, a2, ..., an là các số thực dương thoả mãn a1a2...an = 1 và cho trước n(n ∈ N, n > 2). Chứng minh rằng a1 + 3 (a1 + 1)2 + a2 + 3 (a2 + 1)2 + ... + an + 3 (an + 1)2 ≥ 3. Chứng minh. Ta xét với n = 3. Lúc này bằng cách chọn các số thực dương a, b, c sao cho a a1 = b b1 = c c1 = k ≥ 1; abc = 1. Khi đó ta thấy a1 + 3 (a1 + 1)2 + a2 + 3 (a2 + 1)2 + a3 + 3 (a3 + 1)2 ≥ a + 3 (a + 1)2 + b + 3 (b + 1)2 + c + 3 (c + 1)2 . Đặt a1 = 2 1 + a , b1 = 2 1 + b , c1 = 2 1 + c . Ta phải chứng minh a1 + b1 + c1 + a 2 1 + b 2 1 + c 2 1 ≥ 6. Chú ý rằng với điều kiện abc = 1, cho ta( 1 a1 − 1 2 )( 1 b1 − 1 2 )( 1 c1 − 1 2 ) = abc 8 = 1 8 . Điều đó xảy ra khi và chỉ khi (2− a1)(2− b1)(2− c1) = a1b1c1. Đặt x = a1 − 1, y = b1 − 1, z = c1 − 1, thì x, y, z ≥ −1, nên (x + 1)(y + 1)(z + 1) = (1− x)(1− y)(1− z). Từ đó suy ra x + y + z + xyz = 0. Do 1 ≥ x, y, z ≥ −1 nên x2 + y2 + z2 ≥ 3(xyz)2/3 ≥ 3xyz. 30 Vậy∑ x,y,z x(x + 3) = ∑ sym (a1 − 1)(a2 + 2) = a1 + b1 + c1 + a21 + b21 + c21 − 6 ≥ 0. Như vậy, ta đã chứng minh được bất đẳng thức trong bài toán 4 đúng với n = 3. Trong trường hợp tổng quát, ta chỉ cần chọn 3 số thực dương a1, a2, a3 sao cho a1a2a3 < 1, lúc đó ta dẫn về trường hợp đã xét với n = 3 nhờ bất đẳng thức sau n∑ i=1 ai + 3 (ai + 1)2 ≥ a1 + 3 (a1 + 1)2 + a2 + 3 (a2 + 1)2 + a3 + 3 (a3 + 1)2 . Đó là điều phải chứng minh. 2 Chú ý rằng bất đẳng thức trong bài toán 4, trở thành đẳng thức khi và chỉ ._.

Các file đính kèm theo tài liệu này:

  • pdfLA5839.pdf
Tài liệu liên quan